JEE Main & Advanced Mathematics Statistics Question Bank Mean

  • question_answer
    The A.M. of a 50 set of numbers is 38. If two numbers of the set, namely 55 and 45 are discarded, the A.M. of the remaining set of numbers is                [Kurukshetra CEE 1993]

    A)                 38.5        

    B)                 37.5        

    C)                 36.5        

    D)                 36

    Correct Answer: B

    Solution :

               Given, \[\frac{\Sigma {{x}_{i}}}{50}=38,\,\,\,\therefore \Sigma {{x}_{i}}=1900\]                    New value of \[\Sigma {{x}_{i}}=1900-55-45\]\[=1800\], \[n=48\]                                 \ New mean \[=\frac{1800}{48}\]\[=37.5\].


You need to login to perform this action.
You will be redirected in 3 sec spinner